A baker's bread recipe calls for 3
cups of flour. She plans to make 2 loaves of
bread
If the baker's measuring scoop holds cup, how many scoops of
flour will she need in all?

Answers

Answer 1
answer: 6 cups
explanation: she has 1 cup measuring cup and has to fill it 3 times for each loaf 3*2=6

Related Questions

Mark had 3/2 cans of paint and used 1/2 cans for his room. What fraction of the paint did he use

Help I’m slowww

Answers

Answer:

1/3 fraction of whole paint is used by mark

Step-by-step explanation:

Mark used  1/2 out of 3/2 cans.

[tex]\frac{1}{2}[/tex] ÷ [tex]\frac{3}{2} = \frac{1}{2}*\frac{2}{3}=\frac{1}{3}[/tex]

A rectangular sheet of paper is 12 1/2 cm long and 10 2/3 cm wide.Find it's perimeter​

Answers

Answer:

Given

length of rectangular sheet of paper is 12 (1/2) i.e. (25/2)

Breadth of rectangular sheet of paper is 10 (2/3) i.e. (32/3)

But we know that perimeter of rectangle = 2 (length + breadth)

Perimeter of rectangular sheet = 2 [(25/2) + (32/3)]

LCM of 2 and 3 is 6,

by taking this and simplifying we get

Perimeter = 2[(25 × 3)/6 + (32 × 2)/6]

= 2[(75/6) + (64/6)]

= 2(139/6) = (139/3)

= 46 (1/3) cm

Answer:

Perimeter of rectangular sheet = 2 [(25/2) + (32/3)]

LCM of 2 and 3 is 6,

by taking this and simplifying we get

Perimeter = 2[(25 × 3)/6 + (32 × 2)/6]

= 2[(75/6) + (64/6)]

= 2(139/6) = (139/3)

= 46 (1/3) cm

Step-by-step explanation:

Question 1 (5 points)
Determine the value of x.
3
3V2
6
3V3

Answers

Answer:

Step-by-step explanation:

PLISSSSSSSS HELPPPPPP!!!!!!
i will give brainliest

Answers

0
14
49
49
-14

Hope this helps!!

Evaluate the question in the photo attached please. ASAP

Answers

The answer is 29 I think
The answer is 29

——————————

Explanation:

X^2 = 4^2 when we replace x with 4, 4^2 is 16

And 3x = 3*4 which makes 12

We can then add these sums together to get 28

28 - 7 = 21 and 21 + 8 = 29


Hope this helps, have a great day

The angle of elevation of a tree at a distance of 10m from the foot of the tree is 43°. Find the height of the tree

Answers

Answer:

9.32m is the height of. the tree from the ground.

The value of the expression 10 - 1/2^4 x 48
A = 2
B = 4
C = 5
D = 7

Answers

Answer:

option d is correct answer

Find the length of side xx in simplest radical form with a rational denominator.

Answers

Answer:

Solution given:

it is a right angled isosceles triangle

so

perpendicular [p]=base[b]=3

hypotenuse [h]=x

we have

by using Pythagoras law

p²+b²=h²

3²+3²=h²

18=h²

h=[tex]\sqrt{18}[/tex]

x=[tex]\bold{3\sqrt{2}}[/tex]

x + y = 3, 4y = -4x - 4
System of Equations

Answers

Answer:

no solutions

Step-by-step explanation:

Hi there!

We're given this system of equations:

x+y=3

4y=-4x-4

and we need to solve it (find the point where the lines intersect, as these are linear equations)

let's solve this system by substitution, where we will set one variable equal to an expression containing the other variable, substitute that expression to solve for the variable the expression contains, and then use the value of the solved variable to find the value of the first variable

we'll use the second equation (4y=-4x-4), as there is already only one variable on one side of the equation. Every number is multiplied by 4, so we'll divide both sides by 4

y=-x-1

now we have y set as an expression containing x

substitute -x-1 as y in x+y=3 to solve for x

x+-x-1=3

combine like terms

-1=3

This statement is untrue, meaning that the lines x+y=3 and 4y=-4x-4 won't intersect.

Therefore the answer is no solutions

Hope this helps! :)

The graph below shows the two equations graphed; they are parallel, which means they will never intersect. If they don't intersect, there's no common solution

Please help me w the answer

Answers

Answer:

[tex]\frac{2(x-6)(x-10)}{(x-4)(x-5)}[/tex]

Step-by-step explanation:

In essence, one needs to work their way backwards to solve this problem. Use the information to construct the function.

The function has verticle asymptotes at (x = 4) and (x = 5). This means that the denominator must have (x - 4) and (x - 5) in it. This is because a verticle asymptote indicates that the function cannot have a value at these points, the function jumps at these points. This is because the denominator of a fraction cannot be (0), the values (x - 4) and (x - 5) ensure this. Since if (x) equals (4) or (5) in this situation, the denominator would be (0) because of the zero product property (this states that any number times zero equals zero). So far we have assembled the function as the following:

[tex]\frac{()}{(x-4)(x-5)}[/tex]

The function has x-intercepts at (6, 0), and (0, 10). This means that the numerator must equal (0) when (x) is (6) or (10). Using similar logic that was applied to find the denominator, one can conclude that the numerator must be ([tex](x - 6)(x-10)[/tex]). Now one has this much of the function assembled

[tex]\frac{(x-6)(x-10)}{(x-4)(x-5)}[/tex]

Finally one has to include the y-intercept of (0, 120). Currently, the y-intercept is (60). This is found by multiplying the constants together. (6 * 10) equals (60). One has to multiply this by (2) to get (120). Therefore, one must multiply the numerator by (2) in order to make the y-intercept (120). Thus the final function is the following:

[tex]\frac{2(x-6)(x-10)}{(x-4)(x-5)}[/tex]

please someone explain this

Answers

Answer: 68

Step-by-step explanation: Complementary angles are angles that add up to 90. So, you need to do 90-22=68.

Which equation is the inverse of 2(x – 2)2 = 8(7 + y)?

Answers

Answer:

y = (4x - 71)/8

Step-by-step explanation:

2(x - 2)2 = 8(7 + y) solve for y instead of x for the inverse equation

4x - 8 = 63 + 8y

4x - 8 - 63 = 8y

4x - 71 = 8y

y = (4x - 71)/8

Answer:

A

Step-by-step explanation:


Can someone help solve the problems 2-4

Answers

Answer:

1234567891011121314151617181920

Step-by-step explanation:

you just count

PLEASE I NEED HELP WITH THIS ONE​

Answers

Answer:

H

Step-by-step explanation:

When h=0,t=45.

so we can exclude F.

When h=10,t=15.

only H satisfiy the condition.

Answer:

H

The line shows an inverse proportionality between temperature and time:

[tex]{ \tt{t \: \alpha \: \frac{1}{h} }} \\ \\ { \tt{t = \frac{k}{h} }}[/tex]

Slope or change:

[tex] = \frac{45 - 30}{0 - 5} \\ = - 3[/tex]

y-intercept:

[tex]c = 45[/tex]

General equation:

[tex]y = - 3x + 45[/tex]

n a test,correct answers carry +3 marks and wrong answers carry -1 marks.Ramesh answered all the questions.He scored 79 marks,though he maked 5 mistakes.Find the number of correct answers?

Answers

Answer:

The number of correct answers is 28

Step-by-step explanation:

The value of a correct answer = +3

The value of a wrong answer = -1

The number of questions Ramesh answered = All the questions

The number of mistakes in the question = 5,

Let x represent the number of correct answers therefore, we get;

+3 × x + (-1) × 5 = 79

∴ x = (79 - (-1) × 5)/+3 =  28

The number of correct answers, x = 28

find the value of 5 + 8 / 4 * 3​

Answers

Answer:

44

Step-by-step explanation:

5+8/4*3

5+24/4

20+24

44

Given: triangle RST is circumscribed about circle A.
m∠APT = _____°

Answers

Answer:

90

Step-by-step explanation:

From the given drawing, we have;

ΔRST is circumscribed about circle A

The center of the circle A = The point A

The line RT = A tangent to the circle A

The radius to the circle A = The line AP

According to circle theory, a line which is tangent to a circle is perpendicular to the radius of the circle drawn from the point of tangency

Where two lines are perpendicular to each other, then the angle formed between them = 90°

The angle formed between a tangent and the radius of the circle = m∠APT

Therefore;

m∠APT = 90°



This year, Carlos planted 6 more than one-third of the cucumber plants he planted last year. How many cucumber
plants did he plant this year if last year he planted 12 plants?
06
09
O 10
12

Answers

Answer:

10

Step-by-step explanation:

last year he planted 12.

1/3 of that is 12/3 = 4.

6 more than that is 4 + 6 = 10.

Answer: C.)  10

Step-by-step explanation:

What is the sum of the 15th square number and the 5th cube number?

Answers

The sum of the 15th square number and the 5th cube number is 350.

The 15th square number will be:

15² = 15 × 15

= 225

The 5th cube number will be:

5³ = 5 × 5 × 5

= 125

The sum of the numbers will be:

225 + 125

= 350

Therefore, we get that, the sum of the 15th square number and the 5th cube number is 350.

Learn more about sum here:

https://brainly.com/question/17695139

#SPJ1

Lines L and M are parallel.

Help I’ll make u brainliest if it’s right!!

Answers

Answer:

3 = 142°

Step-by-step explanation:

L // M

∠2 = 38°    {Corresponding angles are congruent}

∠2 + ∠3 = 180   {Linear pair}

38 + ∠3 = 180

       ∠3 = 180 - 38

3 = 142°

the answer is 142 degrees, get 180 degrees from a straight lines and subtract the acute angle from 180 to get the answer, 180-38

There is a bag with only red marbles and blue marbles.
The probability of randomly choosing a red marble is
7/12
There are 84 marbles in total in the bag and each is equally likely to be chosen.
Work out how many red marbles there must be.

Answers

Answer:

49

Step-by-step explanation:

The probability of choosing a red marble is equal to the number of red marbles over the number of total marbles there are.

Therefore, let the number of red marbles be [tex]x[/tex].

We have the following equation:

[tex]\frac{x}{84}=\frac{7}{12}[/tex]

Cross-multiplying, we get:

[tex]12x=7\cdot 84,\\x=\frac{84\cdot 7}{12},\\x=\boxed{49}[/tex]

Therefore, there are 49 red marbles in the bag.

20. It takes Zach 15 minutes to walk 7 blocks to the swimming pool. 7 At this rate, how many blocks can he walk in one minute? Circle the letter of the correct answer. how do I do this step by step to solve it by myself ​

Answers

Answer:

Zach chose C as the correct answer

Uh I think it would be 0.47

7 divided by 15 is 0.47, that’s the amount of miles divided by the number of minutes. I’m really sleep deprived so sorry if this answer is wrong 3

Let $S$ be the set of points $(a,b)$ in the coordinate plane, where each of $a$ and $b$ may be $-1$, 0, or 1. How many distinct lines pass through at least two members of $S$

Answers

Answer:

20 Lines

Step-by-step explanation:

According to the Question,

Given That, Let S be the set of points (a, b) in the coordinate plane, where each of a and b may be -1, 0, or 1.

Now,  the total pairs of points which can be formed is 9

And, the line passing through 2 such points 9c2 = 9! / (2! x 7!) = 9x4 ⇒ 36

Here, We have overcounted all of the lines which pass through three points.

And, each line that passes through three points will have been counted 3c2 = 3! / 2! ⇒ 3 times

Now, the sides of the square consist of 3 points. We have counted each side thrice, so 4*2 are repeated.

Therefore, the distinct lines pass through at least two members of S is 3 horizontal, 3 vertical, and 2 diagonal lines, so the answer is 36 - 2(3+3+2) = 20 Lines

The sum of two numbers is -5 and their difference is -1. Find the two
numbers.

Answers

Answer:

x=-3 and y=-2

Step-by-step explanation:

let the numbers be x and y

x+y=-5

x-y=-1

therefore x=-5-y

-5-y-y=-1

-2y=-1+5

-2y=4

y=-2

×=-5-(-2)

x=-5+2

x=-3

Answer: -2 and -3

Step-by-step explanation:

Number #1 = xNumber #2 = y

x + y = -5

x - y = -1 -> x = y - 1

(y - 1) + y = -5

y - 1 + y = -5

2y = 1 - 5

2y = -4

y = -2

x = y - 1 = -2 - 1 = -3

If you have 6 periods per day at school and math is 1 of them, what percentage of your school day is spent in math?

Answers

Answer:

16.67% of your day is spent in math class.

Step-by-step explanation:

The total would be 100% and then since you have 6 periods we divide 100 by 6 to get 16.67%. So 16.67% of your day is spent in math class.

which eqation represents the line that passes through (-6, 7) and (-3, 6)

Answers

Answer:

The answer is y= - ⅓x + 5 in slope intercept form and y-7 = - ⅓ (x + 6) in point slope form.

Which of the following values of r will result in a true statement when substituted into the given equation?

2(4r + 4) = -16

A. r = -3
B. r = -2
C. r = 2
D. r = 3

Answers

I think that it is A I guessed

A rectangle has a perimeter of 80 cm and its length is 1 cm more than twice its width. Find the dimensions of a rectangle given that its perimeter is 80 cm and its length is 1 cm more than twice its width. Set up your solution using the variables L for the length, W for the width, and P for the perimeter. Part a: Using the definition of the perimeter, write an equation for P in terms of L and W. Part b: Using the relationship given in the problem statement, write an equation for L in terms of W. Solve the equations from parts a and b. Part c: The length is ? Cm. Part d: The width is? Cm.

Answers

Answer: (a) P = 6W + 2

(b) L = 2W + 1

(c) Width = 13cm

Length = 27cm

Step-by-step explanation:

The formula for perimeter of a rectangle is 2(length + width). Since the length is 1 cm more than twice its width, then the length will be:

L = (2 × W) + 1

(b) L = 2W + 1

Therefore, P = 2(L + W)

P = 2( 2W + 1) + 2W

P = 4W + 2 + 2W

(a) P = 6W + 2

Since perimeter is given as 80cm. Therefore,

P = 6W + 2

6W + 2 = 80

6W = 80 - 2

6W = 78

W = 78/6

W = 13

Width is 13cm

Length = 2W + 1

Length = 2(13) + 1

Length = 27cm

The width of the rectangle is 13cm and the length is 27cm.

Description of a rectangle

A rectangle is a quadrilateral. Opposite sides are equal. The four angles in a rectangle is equal to 90 degrees.

The formula for determining the perimeter of a rectangle = 2x (length + width)

P = 2(L + W)

Perimeter = 80 length = 1 + 2w Width = w

Determining the values of width and length

80 = 2(1 + 2w + w)

80 = 2(1 + 3w)

40 = 1 + 3w

40 - 1 = 3w

39 = 3w

w = 13cm

Length = 1 + 2(13) = 27cm

To learn more about rectangles, please check: https://brainly.com/question/16595449

William needs to work out the size of angle Y in this diagram

One of William’s reasons are wrong.

Write down the correct reason.

Answers

Answer:

because internal staggal angles are equal

Step-by-step explanation:

The first reason is wrong.

Angle EGH and DEG are internal staggal angles:

the two angles are on both sides of the cut line EG, and the two angles are between the two divided lines.

{the definition of internal staggal angle}

FOR EASY BRAINLIEST ANSWER QUESTION BELOW!

1. Solve each word problem .twice a number added three times the sum of the number and 2 is more than 17. Find the numbers that satisfy condition

Answers

Answer:

28

Step-by-step explanation:

Other Questions
All rocks follow the same pathway through the rock cycle is a false statement.Why? What is (a) the x component and (b) the y component of the net electric field at the square's center Find x. Simplify completely.1625X =[?] Simultaneous equations 5x-4y=19 x+2y=8 explain the chemical reaction Which of the following is a unifying characteristic of life? What is the effective annual cost of skipping the discount and paying at the end of the net period for the following credit terms: 6/10, net 70 Companies that operate in a ___________________ environment have the lowest amount of environmental uncertainty as illustrated in the environmental uncertainty matrix. Which organism in the food web below is found in the first trophic level of theecosystem? A. Fungus B. Strangler fig C. Fruit bat D. Sloth A coin is tossed repeatedly, and a payoff of 2n dollars is made, where n is the number of the toss on which the first Head appears. So TTH pays $8, TH pays $4 and H pays $2. Write a program to simulate playing the game 10 times. Display the result of the tosses and the payoff. At the end, display the average payoff for the games played. A typical run would be: Ron is pretending to be a monster and chasing his little brother around the house. Which type of play is this What is the general form of the equation for the given circle centered at [0, 0)? Solve. Algebra 11-4p-2p=1-5p In golf, scores are often written in relationship to par, the average score for a round at a certain course. Write an integer to represent a score that is 7 under par. Consider the market for purple potatoes below and assume that a price ceiling of $30 is imposed by the government. Calculate the deadweight loss: this is so confusing can anyone help? Drag the tiles to the correct boxes to complete the pairs.Match each scenario to the climatic factor that caused it. How does religion impact art? The quadratic function f(x) = -x2 - 6x - 8 is graphed.What are the solutions of the quadratic equation 0 =x2 - 6x - 8?2O 2 and 40-2 and 40-2 and 4O: 2 and 46 5 A-3-1X-2 You are calculating the volume quantity needed for material that will compact to 95% of its original undisturbed volume. You know that you will need to fill a void with a volume of 1,487 cubic yards. What is the volume of material needed from the borrow pit to equal this amount when fully compacted